Diễn Đàn MathScopeDiễn Đàn MathScope
  Diễn Đàn MathScope
Ghi Danh Hỏi/Ðáp Community Lịch

Go Back   Diễn Đàn MathScope > Sơ Cấp > Đại Số và Lượng Giác > Các Bài Toán Đã Được Giải

News & Announcements

Ngoài một số quy định đã được nêu trong phần Quy định của Ghi Danh , mọi người tranh thủ bỏ ra 5 phút để đọc thêm một số Quy định sau để khỏi bị treo nick ở MathScope nhé !

* Nội quy MathScope.Org

* Một số quy định chung !

* Quy định về việc viết bài trong diễn đàn MathScope

* Nếu bạn muốn gia nhập đội ngũ BQT thì vui lòng tham gia tại đây

* Những câu hỏi thường gặp

* Về việc viết bài trong Box Đại học và Sau đại học


Trả lời Gởi Ðề Tài Mới
 
Ðiều Chỉnh Xếp Bài
Old 02-09-2009, 10:32 AM   #16
Hung_DHSP
+Thành Viên Danh Dự+
 
Tham gia ngày: Sep 2008
Đến từ: K42 CSP K53 Kinh tế quốc dân
Bài gởi: 223
Thanks: 28
Thanked 86 Times in 63 Posts
From Mathlinks

Bài toán 10:Cho $x,y,z\geq 0 $.Tìm giá trị nhỏ nhất của:
$P=\frac{x^{7}z}{x^{5}y^{2}z+2y^{6}}+\frac{y^{7}z^{ 6}}{y^{5}z^{4}+2x}+\frac{1}{z^{2}x^{2}+2x^{6}yz^{7 }} $
[RIGHT][I][B]Nguồn: MathScope.ORG[/B][/I][/RIGHT]
 
Hung_DHSP is offline   Trả Lời Với Trích Dẫn
Old 02-09-2009, 11:45 AM   #17
caube94
+Thành Viên+
 
caube94's Avatar
 
Tham gia ngày: Nov 2008
Đến từ: Gia Lâm -Hà Nội
Bài gởi: 117
Thanks: 9
Thanked 38 Times in 26 Posts
Trích:
Nguyên văn bởi trungdeptrai View Post
Bài 9(MOP 02)Với a,b,c dương.CM:
${\left(\frac{2a}{b+c} \right)}^{\frac{2}{3}}+{\left(\frac{2b}{c+a} \right)}^{\frac{2}{3}}+{\left(\frac{2c}{a+b} \right)}^{\frac{2}{3}}\geq 3 $
Chém tạm
9.$LHS^3[4\sum a^2(b+c)^2] \ge 16(a+b+c)^4 $
Nên ta cần cm:
$16(a+b+c)^4 \ge 108\sum a^2(b+c)^2 $
$<=>3\sum a^4+(a^2+b^2+c^2)^2-6abc(a+b+c)+16\sum bc(b-c)^2 \ge 0 $
It's true
2. Theo cách giải này cho gọn
$LHS^2[\sum a(a^2+8bc)] \ge (a+b+c)^3 $
Mà:
$(a+b+c)^3 \ge \sum a(a^2+8bc) $
[RIGHT][I][B]Nguồn: MathScope.ORG[/B][/I][/RIGHT]
 
__________________
Ðừng khóc vì mọi việc đã qua, hãy cười vì mọi việc đang chờ phía trước.

thay đổi nội dung bởi: caube94, 02-09-2009 lúc 12:04 PM
caube94 is offline   Trả Lời Với Trích Dẫn
The Following 2 Users Say Thank You to caube94 For This Useful Post:
searcher (02-09-2009)
Old 02-09-2009, 02:21 PM   #18
xiloxila
+Thành Viên+
 
xiloxila's Avatar
 
Tham gia ngày: Apr 2009
Đến từ: Lỗ đen của vũ trụ
Bài gởi: 52
Thanks: 19
Thanked 9 Times in 8 Posts
Bài toán 11 Cho a,b,c là các số thực không âm tùy ý chứng minh
$\frac{a^3}{(a+b)^3}+\frac{b^3}{(c+a)^3}+\frac{c^3} {(c+a)^3}+\frac{5abc}{(a+b)(b+c)(c+a)}\geq 1 $
[RIGHT][I][B]Nguồn: MathScope.ORG[/B][/I][/RIGHT]
 

thay đổi nội dung bởi: xiloxila, 03-09-2009 lúc 10:19 AM
xiloxila is offline   Trả Lời Với Trích Dẫn
The Following User Says Thank You to xiloxila For This Useful Post:
tho 2010 (20-03-2010)
Old 02-09-2009, 04:10 PM   #19
Red Devils
+Thành Viên+
 
Red Devils's Avatar
 
Tham gia ngày: Aug 2009
Đến từ: Lớp 55CLC2, trường ĐHXD
Bài gởi: 205
Thanks: 28
Thanked 395 Times in 82 Posts
Trích:
Nguyên văn bởi Conan Edogawa View Post
Bài toán 2: (IMO Shortlist 2001)
Chứng minh rắng với mọi số thực dương $a, b, c $ ta có:
$\frac{a}{\sqrt{a^{2}+8bc}}+\frac{b}{\sqrt{b^{2}+8c a}}+\frac{c}{\sqrt{c^{2}+8ab}}\geq 1 $
Lời giải: Chuẩn hóa $abc=1 $. Khi đó BĐT cần cm tương đương:$\frac{1}{\sqrt{1+8a}}+\frac{1}{\sqrt{1+8b}}+\frac{ 1}{\sqrt{1+8c}}\ge 1 $

$\Leftrightarrow \sum{\sqrt{(1+8a)(1+8b)}}\ge \sqrt{(1+8a)(1+8b)(1+8c)} $

$\Leftrightarrow 8(a+b+c)+2\sqrt{(1+8a)(1+8b)(1+8c)}\sum{\sqrt{1+8a }\ge 510} $

Do $abc=1\Rightarrow a+b+c\ge 3 $ và $(1+8a)(1+8b)(1+8c)\ge 9{{a}^{\frac{8}{9}}}{{b}^{\frac{8}{9}}}{{c}^{\frac {8}{9}}}=729 $

Và $\sum{\sqrt{1+8a}\ge \sum{\sqrt{9{{a}^{\frac{8}{9}}}}\ge 9{{\left( abc \right)}^{\frac{4}{27}}}}=9} $

Cộng tất cả lại ta được đpcm.
Thật ra đoạn đầu nên viết lại là:
Bất đẳng thức cần chứng minh tương đương với:
$\frac{1}{\sqrt{1+8\frac{bc}{a^2}}}+\frac{1}{\sqrt{ 1+8\frac{ca}{b^2}}}+\frac{1}{\sqrt{1+8\frac{ab}{c^ 2}}}\geq1 $
Đặt $a_1=\frac{bc}{a^2}, b_1=\frac{ca}{b^2},c_1=\frac{ab}{c^2} $ suy ra $a_1b_1c_1=1 $ và BĐT cần chứng minh trở thành:
$\frac{1}{\sqrt{1+8a_1}}+\frac{1}{\sqrt{1+8b_1}}+ \frac{1}{\sqrt{1+8c_1}} \ge 1 $

Trích:
Nguyên văn bởi Hung_DHSP View Post
Bài toán 10: (Đề thi chọn Đội tuyển KHTN Vòng 2 năm học 2008-2009) Cho $x,y,z\geq 0 $.Tìm giá trị nhỏ nhất của:
$P=\frac{x^{7}z}{x^{5}y^{2}z+2y^{6}}+\frac{y^{7}z^{ 6}}{y^{5}z^{4}+2x}+\frac{1}{z^{2}x^{2}+2x^{6}yz^{7 }} $
Bài này ko đẹp mắt lắm, nhìn dạng là biết ngay sau 1 vài bước đặt để đổi biến sẽ đưa về BĐT đối xứng. Đây là đề thi Chọn Đội tuyển vòng 2 của KHTN năm 2008. Xem thêm tại đây: [Only registered and activated users can see links. ]
Trích:
Nguyên văn bởi xiloxila View Post
Bài toán 11: Cho $a,b,c $ là các số thực không âm tùy ý chứng minh
$\frac{a^3}{(a+b)^3}+\frac{b^3}{(c+a)^3}+\frac{c^3} {(c+a)^3}+\frac{5abc}{(a+b)(b+c)(c+a)}\geq 1 $
Bài này trong Sáng tạo BĐT chứng minh sử dụng hằng đẳng thức, nếu minh nhớ ko nhầm thì trong cuốn BĐT Suy luận và khám phá có 1 đánh giá mạnh hơn là:
$\frac{a^3}{(a+b)^3}+\frac{b^3}{(c+a)^3}+\frac{c^3} {(c+a)^3}+\frac{5abc}{(a+b)(b+c)(c+a)}\geq \frac{a^2+b^2+c^2}{ab+bc+ca} $
Vì không có sách trong tay nên mình cũng ko dám dám chắc 100%.

P/S: Mong các bạn ghi rõ nguồn của bài toán vì BĐT rất mênh mông, Topic chỉ ghi lại những BĐT trong các kì thi Olympiad chứ ko rải rác mặc dù có thể có những BĐT rất đẹp nhưng chưa từng xuất hiện trong kì thi nào.
[RIGHT][I][B]Nguồn: MathScope.ORG[/B][/I][/RIGHT]
 
Red Devils is offline   Trả Lời Với Trích Dẫn
The Following 2 Users Say Thank You to Red Devils For This Useful Post:
Conan Edogawa (02-09-2009)
Old 02-09-2009, 06:40 PM   #20
Conan Edogawa
+Thành Viên+
 
Conan Edogawa's Avatar
 
Tham gia ngày: Sep 2008
Đến từ: Trường ĐH Kinh tế TP.HCM
Bài gởi: 397
Thanks: 136
Thanked 303 Times in 150 Posts
Trích:
Nguyên văn bởi trungdeptrai View Post
Bài 9(MOP 02)
Với a,b,c dương.CM:
${\left(\frac{2a}{b+c} \right)}^{\frac{2}{3}}+{\left(\frac{2b}{c+a} \right)}^{\frac{2}{3}}+{\left(\frac{2c}{a+b} \right)}^{\frac{2}{3}}\geq 3 $
Trích:
Nguyên văn bởi caube94 View Post
Chém tạm
9.$LHS^3[4\sum a^2(b+c)^2] \ge 16(a+b+c)^4 $
Nên ta cần cm:
$16(a+b+c)^4 \ge 108\sum a^2(b+c)^2 $
$<=>3\sum a^4+(a^2+b^2+c^2)^2-6abc(a+b+c)+16\sum bc(b-c)^2 \ge 0 $
It's true
Cách này ngắn gọn hơn nè(mà bài này cũng là USAMO Summer Program-2002)
Áp dụng AM-GM cho 3 số dương:

$1+\frac{b+c}{2a}+\frac{b+c}{2a}\ge 3\sqrt[3]{{{\left( \frac{b+c}{2a} \right)}^{2}}}\Rightarrow \frac{a+b+c}{a}\ge 3{{\left( \frac{b+c}{2a} \right)}^{\frac{2}{3}}} $

$\Rightarrow {{\left( \frac{2a}{b+c} \right)}^{\frac{2}{3}}}\ge \frac{3a}{a+b+c}\Rightarrow \sum{{{\left( \frac{2a}{b+c} \right)}^{\frac{2}{3}}}}\ge \frac{3(a+b+c)}{a+b+c}=3 $
------------------------------
Bài 1: (IMO Shortlist 2001)
Cho $n $ số thực tuỳ ý $x_{1},x_{2},\ldots,x_{n} $. Chứng minh rằng:
$\frac{x_{1}}{1+x_{1}^{2}}+\frac{x_{2}}{1+x_{1}^{2} +x_{2}^{2}}+\cdots+\frac{x_{n}}{1+x_{1}^{2}+\cdots +x_{n}^{2}}<\sqrt{n} $

Bài 4: (14th Turkish Mathematical Olympiad, 2006)
Với mọi số thực dương $a, b, c $ thỏa mãn đẳng thức $a+b + c=1 $. Chứng minh rằng:
$\frac{1}{ab+2c^2+2c}+\frac{1}{bc+2a^2+2a}+\frac{1} {ca+2b^2+2b}\geq \frac{1}{ab+bc+ca} $

Bài 12: (USAMO Summer Program – 2006)
Cho $a,b,c>0 $. Chứng minh:

$\frac{1}{a(1+b)}+\frac{1}{b(1+c)}+\frac{1}{c(1+a)} \ge \frac{3}{\sqrt[3]{abc}(1+\sqrt[3]{abc})} $

Bài 13: (IMO Shortlist -2004)
Cho $a,b,c>0 $ thỏa $ab+bc+ca=1 $. Chứng minh:

$\sqrt[3]{\frac{1}{a}+6b}+\sqrt[3]{\frac{1}{b}+6c}+\sqrt[3]{\frac{1}{c}+6a}\le \frac{1}{abc} $

Bài 14: (Romania – 2005)
Cho $a,b,c>0 $ thòa $abc\ge 1 $. Chứng minh:

$\frac{1}{1+a+b}+\frac{1}{1+b+c}+\frac{1}{1+c+a}\le 1 $
[RIGHT][I][B]Nguồn: MathScope.ORG[/B][/I][/RIGHT]
 

thay đổi nội dung bởi: Conan Edogawa, 02-09-2009 lúc 07:02 PM Lý do: Tự động gộp bài
Conan Edogawa is offline   Trả Lời Với Trích Dẫn
The Following 4 Users Say Thank You to Conan Edogawa For This Useful Post:
Red Devils (03-09-2009), searcher (03-09-2009), tho 2010 (20-03-2010)
Old 02-09-2009, 07:37 PM   #21
caube94
+Thành Viên+
 
caube94's Avatar
 
Tham gia ngày: Nov 2008
Đến từ: Gia Lâm -Hà Nội
Bài gởi: 117
Thanks: 9
Thanked 38 Times in 26 Posts
Trích:
Nguyên văn bởi Conan Edogawa View Post
Cách này ngắn gọn hơn nè(mà bài này cũng là USAMO Summer Program-2002)
Áp dụng AM-GM cho 3 số dương:

$1+\frac{b+c}{2a}+\frac{b+c}{2a}\ge 3\sqrt[3]{{{\left( \frac{b+c}{2a} \right)}^{2}}}\Rightarrow \frac{a+b+c}{a}\ge 3{{\left( \frac{b+c}{2a} \right)}^{\frac{2}{3}}} $

$\Rightarrow {{\left( \frac{2a}{b+c} \right)}^{\frac{2}{3}}}\ge \frac{3a}{a+b+c}\Rightarrow \sum{{{\left( \frac{2a}{b+c} \right)}^{\frac{2}{3}}}}\ge \frac{3(a+b+c)}{a+b+c}=3 $
------------------------------
Bài 1: (IMO Shortlist 2001)
Cho $n $ số thực tuỳ ý $x_{1},x_{2},\ldots,x_{n} $. Chứng minh rằng:
$\frac{x_{1}}{1+x_{1}^{2}}+\frac{x_{2}}{1+x_{1}^{2} +x_{2}^{2}}+\cdots+\frac{x_{n}}{1+x_{1}^{2}+\cdots +x_{n}^{2}}<\sqrt{n} $

Bài 4: (14th Turkish Mathematical Olympiad, 2006)
Với mọi số thực dương $a, b, c $ thỏa mãn đẳng thức $a+b + c=1 $. Chứng minh rằng:
$\frac{1}{ab+2c^2+2c}+\frac{1}{bc+2a^2+2a}+\frac{1} {ca+2b^2+2b}\geq \frac{1}{ab+bc+ca} $

Bài 12: (USAMO Summer Program – 2006)
Cho $a,b,c>0 $. Chứng minh:

$\frac{1}{a(1+b)}+\frac{1}{b(1+c)}+\frac{1}{c(1+a)} \ge \frac{3}{\sqrt[3]{abc}(1+\sqrt[3]{abc})} $

Bài 13: (IMO Shortlist -2004)
Cho $a,b,c>0 $ thỏa $ab+bc+ca=1 $. Chứng minh:

$\sqrt[3]{\frac{1}{a}+6b}+\sqrt[3]{\frac{1}{b}+6c}+\sqrt[3]{\frac{1}{c}+6a}\le \frac{1}{abc} $

Bài 14: (Romania – 2005)
Cho $a,b,c>0 $ thòa $abc\ge 1 $. Chứng minh:

$\frac{1}{1+a+b}+\frac{1}{1+b+c}+\frac{1}{1+c+a}\le 1 $
Bài 1;4 anh post lại à
Làm đc bài nào post lun

Bài 12:
Ta có:
$LHS^2 \ge \sum \frac{1}{ab(1+b)(1+c) $
Đến đây đặt:
$\sqrt[3]{abc}=k $ là xong
------------------------------
Tiếp bài 13:
Nó đơn giản chỉ là côsi điểm rơi
$\sqrt[3]{27(\frac{1}{a}+6b)} \le \frac{\frac{1}{a}+6b+3\sqrt{3}+3\sqrt{3}}{3} $
Tương tự cộng lại là xong
------------------------------
------------------------------
Đến bài 14 nó cũng chỉ dùng đến Am-Gm
$(a+b)(b+c)(c+a) \ge 2(a+b+c)+2 $
Thay bởi điều khác chắc đẹp hơn
[RIGHT][I][B]Nguồn: MathScope.ORG[/B][/I][/RIGHT]
 
__________________
Ðừng khóc vì mọi việc đã qua, hãy cười vì mọi việc đang chờ phía trước.

thay đổi nội dung bởi: caube94, 02-09-2009 lúc 07:57 PM Lý do: Tự động gộp bài
caube94 is offline   Trả Lời Với Trích Dẫn
The Following 3 Users Say Thank You to caube94 For This Useful Post:
Conan Edogawa (02-09-2009), tho 2010 (20-03-2010)
Old 02-09-2009, 09:06 PM   #22
searcher
+Thành Viên+
 
Tham gia ngày: Apr 2009
Bài gởi: 9
Thanks: 5
Thanked 2 Times in 2 Posts
Trích:
2. Theo cách giải này cho gọn
$LHS^2[\sum a(a^2+8bc)] \ge (a+b+c)^3 $
Mà:
$(a+b+c)^3 \ge \sum a(a^2+8bc) $
don't understand, please tell me why?
[RIGHT][I][B]Nguồn: MathScope.ORG[/B][/I][/RIGHT]
 
searcher is offline   Trả Lời Với Trích Dẫn
The Following User Says Thank You to searcher For This Useful Post:
tho 2010 (20-03-2010)
Old 02-09-2009, 09:07 PM   #23
caube94
+Thành Viên+
 
caube94's Avatar
 
Tham gia ngày: Nov 2008
Đến từ: Gia Lâm -Hà Nội
Bài gởi: 117
Thanks: 9
Thanked 38 Times in 26 Posts
Trích:
Nguyên văn bởi searcher View Post
2. Theo cách giải này cho gọn
$LHS^2[\sum a(a^2+8bc)] \ge (a+b+c)^3 $
Mà:
$(a+b+c)^3 \ge \sum a(a^2+8bc) $


Thì nó chính là bđt holder
Bạn ko hiểu chỗ nào vậy
[RIGHT][I][B]Nguồn: MathScope.ORG[/B][/I][/RIGHT]
 
__________________
Ðừng khóc vì mọi việc đã qua, hãy cười vì mọi việc đang chờ phía trước.
caube94 is offline   Trả Lời Với Trích Dẫn
The Following User Says Thank You to caube94 For This Useful Post:
tho 2010 (20-03-2010)
Old 02-09-2009, 09:33 PM   #24
Minh Tuấn
+Thành Viên Danh Dự+
 
Tham gia ngày: Jul 2008
Bài gởi: 218
Thanks: 13
Thanked 78 Times in 41 Posts
Trích:
Nguyên văn bởi Red Devils View Post
Bài toán 4: (14th Turkish Mathematical Olympiad, 2006)
Với mọi số thực dương $a, b, c $ thỏa mãn đẳng thức $a+b + c=1 $. Chứng minh rằng:
$\frac{1}{ab+2c^2+2c}+\frac{1}{bc+2a^2+2a}+\frac{1} {ca+2b^2+2b}\geq \frac{1}{ab+bc+ca} $
Lời giải:
Ta có:$VT = \sum {\frac{{{a^2}{b^2}}}{{{a^3}{b^3} + 2{a^2}{b^2}{c^2} + 2{a^2}{b^2}c}}} \ge \frac{{{{\left( {\sum {ab} } \right)}^2}}}{{\sum {{a^3}{b^3} + 6{a^2}{b^2}{c^2} + 2abc\sum {ab} } }} $
Cần chứng minh:
${\left( {\sum {ab} } \right)^3} \ge \sum {{a^3}{b^3} + 6{a^2}{b^2}{c^2} + 2abc\sum {ab} } $
$\Leftrightarrow 3abc(a + b)(b + c)(c + a) \ge 6{a^2}{b^2}{c^2} + 2abc(ab + bc + ca) $
Bất đẳng thức cuối đúng vì $\frac{3}{4}abc(a + b)(b + c)(c + a) \ge 6{a^2}{b^2}{c^2} $ và
$\frac{9}{4}abc(a + b)(b + c)(c + a) \ge 2abc(ab + bc + ca) $
$ \Leftrightarrow 9(a + b)(b + c)(c + a) \ge 8(ab + bc + ca) $
$ \Leftrightarrow 9(1 - c)(1 - a)(1 - b) \ge 8(ab + bc + ca) $
$ \Leftrightarrow ab + bc + ca \ge 9abc{\rm{ (*)}} $
$(*) $ đúng vì $ab + bc + ca \ge 3\sqrt[3]{{{{(abc)}^2}}}{\rm{ }} \ge 9abc{\rm{ }} $, do $\sqrt[3]{{abc}} \le \frac{{a + b + c}}{3} = \frac{1}{3}. $
Vậy bất đẳng thức được chứng minh.
[RIGHT][I][B]Nguồn: MathScope.ORG[/B][/I][/RIGHT]
 
Minh Tuấn is offline   Trả Lời Với Trích Dẫn
The Following 3 Users Say Thank You to Minh Tuấn For This Useful Post:
Red Devils (03-09-2009), tho 2010 (20-03-2010)
Old 02-09-2009, 09:49 PM   #25
caube94
+Thành Viên+
 
caube94's Avatar
 
Tham gia ngày: Nov 2008
Đến từ: Gia Lâm -Hà Nội
Bài gởi: 117
Thanks: 9
Thanked 38 Times in 26 Posts
Trích:
Nguyên văn bởi Minh Tuấn View Post
Lời giải:
Ta có:$VT = \sum {\frac{{{a^2}{b^2}}}{{{a^3}{b^3} + 2{a^2}{b^2}{c^2} + 2{a^2}{b^2}c}}} \ge \frac{{{{\left( {\sum {ab} } \right)}^2}}}{{\sum {{a^3}{b^3} + 6{a^2}{b^2}{c^2} + 2abc\sum {ab} } }} $
Cần chứng minh:
${\left( {\sum {ab} } \right)^3} \ge \sum {{a^3}{b^3} + 6{a^2}{b^2}{c^2} + 2abc\sum {ab} } $
$\Leftrightarrow 3abc(a + b)(b + c)(c + a) \ge 6{a^2}{b^2}{c^2} + 2abc(ab + bc + ca) $
Bất đẳng thức cuối đúng vì $\frac{3}{4}abc(a + b)(b + c)(c + a) \ge 6{a^2}{b^2}{c^2} $ và
$\frac{9}{4}abc(a + b)(b + c)(c + a) \ge 2abc(ab + bc + ca) $
$ \Leftrightarrow 9(a + b)(b + c)(c + a) \ge 8(ab + bc + ca) $
$ \Leftrightarrow 9(1 - c)(1 - a)(1 - b) \ge 8(ab + bc + ca) $
$ \Leftrightarrow ab + bc + ca \ge 9abc{\rm{ (*)}} $
$(*) $ đúng vì $ab + bc + ca \ge 3\sqrt[3]{{{{(abc)}^2}}}{\rm{ }} \ge 9abc{\rm{ }} $, do $\sqrt[3]{{abc}} \le \frac{{a + b + c}}{3} = \frac{1}{3}. $
Vậy bất đẳng thức được chứng minh.
Nice solution
Bài 15Bài này cũng khá đẹp mắt;Iran TST 2008)
Cho:$a;b;c \ge 0;ab+bc+ca=1 $.CM:
$\sqrt{a^3+a}+\sqrt{b^3+b}+\sqrt{c^3+c} \ge 2\sqrt{a+b+c} $
[RIGHT][I][B]Nguồn: MathScope.ORG[/B][/I][/RIGHT]
 
__________________
Ðừng khóc vì mọi việc đã qua, hãy cười vì mọi việc đang chờ phía trước.
caube94 is offline   Trả Lời Với Trích Dẫn
The Following 2 Users Say Thank You to caube94 For This Useful Post:
tho 2010 (20-03-2010)
Old 02-09-2009, 10:11 PM   #26
Minh Tuấn
+Thành Viên Danh Dự+
 
Tham gia ngày: Jul 2008
Bài gởi: 218
Thanks: 13
Thanked 78 Times in 41 Posts
Trích:
Nguyên văn bởi Conan Edogawa View Post
Bài 13: (IMO Shortlist -2004)
Cho $a,b,c>0 $ thỏa $ab+bc+ca=1 $. Chứng minh:
$\sqrt[3]{\frac{1}{a}+6b}+\sqrt[3]{\frac{1}{b}+6c}+\sqrt[3]{\frac{1}{c}+6a}\le \frac{1}{abc} $
Áp dụng BĐT Holder cho vế trái ta có:
$\left(\sqrt[3]{\frac{1}{a}+6b}+\sqrt[3]{\frac{1}{b}+6c}+\sqrt[3]{\frac{1}{c}+6a}\right)^3\leq (1+6ab+1+6bc+1+6ca)(\frac{1}{a}+\frac{1}{b}+\frac{ 1}{c})(1+1+1)=\frac{27}{abc}=\frac{27ab.bc.ca}{a^3 b^3c^3}\leq \frac{1}{(abc)^3} $

Bài toán 16: (China TST 2005)
Cho $a, b, c>0 $, $ab+bc+ca=\frac{1}{3} $. Chứng minh rằng:
$\frac{1}{{{a^2} - bc + 1}} + \frac{1}{{{b^2} - ca + 1}} + \frac{1}{{{c^2} - ab + 1}} \le 3 $
Bài toán 17: (China TST 2006)
Cho $x, y, z>0 $ và $x+y+z=1 $. Chứng minh rằng:
$\frac{{xy}}{{\sqrt {xy + yz} }} + \frac{{yz}}{{\sqrt {yz + zx} }} + \frac{{zx}}{{\sqrt {zx + xy} }} \le \frac{{\sqrt 2 }}{2} $
[RIGHT][I][B]Nguồn: MathScope.ORG[/B][/I][/RIGHT]
 

thay đổi nội dung bởi: Minh Tuấn, 02-09-2009 lúc 10:16 PM
Minh Tuấn is offline   Trả Lời Với Trích Dẫn
The Following 2 Users Say Thank You to Minh Tuấn For This Useful Post:
tho 2010 (20-03-2010)
Old 03-09-2009, 02:45 AM   #27
trungdeptrai
+Thành Viên+
 
trungdeptrai's Avatar
 
Tham gia ngày: Oct 2008
Đến từ: Trường THPT Chuyên ĐHSP HN
Bài gởi: 100
Thanks: 12
Thanked 53 Times in 27 Posts
Gửi tin nhắn qua Yahoo chát tới trungdeptrai
Icon10

Bài 18:(IMO shortlist 1998)
Cho x,y,z dương và xyz=1.CM:
$\frac{{x}^{3}}{\left(1+y \right)\left(1+z \right)}+\frac{{y}^{3}}{\left(1+z \right)\left(1+x \right)}+\frac{{z}^{3}}{\left(1+x \right)\left(1+y \right)}\geq \frac{3}{4} $
[RIGHT][I][B]Nguồn: MathScope.ORG[/B][/I][/RIGHT]
 
__________________
Live for Maths - love Maths forever
Nếu được sống thêm một cuộc đời nữa, tôi sẽ lại làm Toán...
trungdeptrai is offline   Trả Lời Với Trích Dẫn
The Following User Says Thank You to trungdeptrai For This Useful Post:
tho 2010 (20-03-2010)
Old 03-09-2009, 07:48 AM   #28
caube94
+Thành Viên+
 
caube94's Avatar
 
Tham gia ngày: Nov 2008
Đến từ: Gia Lâm -Hà Nội
Bài gởi: 117
Thanks: 9
Thanked 38 Times in 26 Posts
Trích:
Nguyên văn bởi trungdeptrai View Post
Bài 18:(IMO shortlist 1998)
Cho x,y,z dương và xyz=1.CM:
$\frac{{x}^{3}}{\left(1+y \right)\left(1+z \right)}+\frac{{y}^{3}}{\left(1+z \right)\left(1+x \right)}+\frac{{z}^{3}}{\left(1+x \right)\left(1+y \right)}\geq \frac{3}{4} $
18.Lại 1 bài điểm rơi
$\frac{x^3}{(1+y)(1+z)}+\frac{1+y}{8}+\frac{1+z}{8} \ge \frac{3}{4}x $
Tương tự cộng lại
[RIGHT][I][B]Nguồn: MathScope.ORG[/B][/I][/RIGHT]
 
__________________
Ðừng khóc vì mọi việc đã qua, hãy cười vì mọi việc đang chờ phía trước.
caube94 is offline   Trả Lời Với Trích Dẫn
The Following User Says Thank You to caube94 For This Useful Post:
tho 2010 (20-03-2010)
Old 03-09-2009, 11:36 AM   #29
Red Devils
+Thành Viên+
 
Red Devils's Avatar
 
Tham gia ngày: Aug 2009
Đến từ: Lớp 55CLC2, trường ĐHXD
Bài gởi: 205
Thanks: 28
Thanked 395 Times in 82 Posts
Trích:
Nguyên văn bởi trungdeptrai View Post
Bài 18:(IMO shortlist 1998)
Cho x,y,z dương và xyz=1.CM:
$\frac{{x}^{3}}{\left(1+y \right)\left(1+z \right)}+\frac{{y}^{3}}{\left(1+z \right)\left(1+x \right)}+\frac{{z}^{3}}{\left(1+x \right)\left(1+y \right)}\geq \frac{3}{4} $
Tổng quát: Với $x, y, z $ là các số thực dương có tích bằng 1, $n $ là số nguyên không nhỏ hơn 3. Chứng minh rằng:
$A = \frac{{{x^n}}}{{(1 + y)(1 + z)}} + \frac{{{y^n}}}{{(1 + x)(1 + z)}} + \frac{{{z^n}}}{{(1 + x)(1 + y)}} \ge \frac{3}{4} $
[RIGHT][I][B]Nguồn: MathScope.ORG[/B][/I][/RIGHT]
 
Red Devils is offline   Trả Lời Với Trích Dẫn
Old 03-09-2009, 04:46 PM   #30
toanlc_gift
+Thành Viên+
 
toanlc_gift's Avatar
 
Tham gia ngày: Jan 2009
Đến từ: FU
Bài gởi: 171
Thanks: 31
Thanked 142 Times in 80 Posts
Icon10 Gần giống VNTST2006

cho a;b;c là 3 số dương thỏa mãn
$3mid\{ a;b;c{\rm{\} }} \ge a + b + c $
chứng minh rằng:
$(a + b + c)(\frac{1}{a} + \frac{1}{b} + \frac{1}{c}) \ge 6\left( {\frac{a}{{b + c}} + \frac{b}{{a + c}} + \frac{c}{{a + b}}} \right) $
[RIGHT][I][B]Nguồn: MathScope.ORG[/B][/I][/RIGHT]
 
toanlc_gift is offline   Trả Lời Với Trích Dẫn
Trả lời Gởi Ðề Tài Mới

Bookmarks

Tags
bất đẳng thức, inequalities


Quuyền Hạn Của Bạn
You may not post new threads
You may not post replies
You may not post attachments
You may not edit your posts

BB code is Mở
Smilies đang Mở
[IMG] đang Mở
HTML đang Tắt

Chuyển đến


Múi giờ GMT. Hiện tại là 10:01 AM.


Powered by: vBulletin Copyright ©2000-2024, Jelsoft Enterprises Ltd.
Inactive Reminders By mathscope.org
[page compression: 119.54 k/136.99 k (12.73%)]